8 ( 11 - 2b ) = -4 ( 4b - 22 )

Answers

Answer 1

Problem

8 ( 11 - 2b ) = -4 ( 4b - 22 )

Solution

We can distribute the terms in the equation and we got:

88 -16b = -16b +88

If we add 16b in boh sides we got:

88 =88

Then for this case we can conclude that this equation has infinite solutions


Related Questions

calculate the length of side AC​

Answers

Answer:

×=12+5

×=144+25

×=169

×=13

A triangle has sides with lengths of 12 yard, 13 yards, and 15 yards. Which numbers are representing the legs of a triangle?

Answers

ANSWER

None of these sides represent the legs.

EXPLANATION

The legs of a triangle are the sides that form the right angle in a right triangle. The legs are always the shortest sides, while the hypotenuse is the longest side.

In this case, the hypotenuse would be 15 yards, while the legs would be 12 yards and 13 yards. For this to be a right triangle, the Pythagorean Theorem must be satisfied,

[tex]\begin{gathered} 12^2+13^2=15^2 \\ 144+169=15^2 \\ 313=225\to false \end{gathered}[/tex]

These side lengths do not satisfy the Pythagorean Theorem and, therefore, this is not a right triangle. If it is not a right triangle, then it does not have legs. Hence, none of these represent the legs.

Solve for y:5x-8y=40

Answers

Solve for y means we need to isolate y from the equation:

We need to use inverse operations to solve equations:

[tex]\begin{gathered} 5x-8y=40 \\ -8y=40-5x \\ y=\frac{-5}{-8}x+\frac{40}{-8} \\ y=\frac{5}{8}x-5 \end{gathered}[/tex]

Add.(7g + 4) + (8g + 2)

Answers

We have to add the expression.

We will group the similar terms:

[tex]\begin{gathered} \mleft(7g+4\mright)+(8g+2) \\ 7g+8g+4+2 \\ 15g+6 \end{gathered}[/tex]

Answer: 15g+6

The number line below represents which combined inequality? xs-6 orx 25 xs -6 and x 2 5 X2 -6 and x s 5 x2-6 or x s 5

Answers

Answer

Option C is correct.

x ≥ -6 and x ≤ 5

Explanation

In graphing inequality equations, the first thing to note is that whenever the equation to be graphed has (< or >), the circle at the beginning of the arrow is usually unshaded.

But whenever the inequality has either (≤ or ≥), the circle at the beginning of the arrow will be shaded.

Since the beginning of the blue mark is a shaded circle, the inequality is (≤ or ≥).

And considering that the region of the answer is between -6 and 5, it is evident that x is greater than or equal to -6 and less than or equal to 5. In mathematical terms,

x ≥ -6 and x ≤ 5

-6 ≤ x ≤ 5

Hope this Helps!!!

Write logaa=4x in exponential form and find x to evaluate logaa for any a>0, a≠1.

Answers

Given:

[tex]\log _aa=4x[/tex]

To find the exponential form of the above, all we need to do is to raise the base a to the power of 4x.

That is;

[tex]a^{4x}=a[/tex]

To find the value of x, we need to raise the power of the right - hand side so that we can equate the exponent

That is;

[tex]a^{4x}=a^1[/tex]

4x = 1

Divide both-side by 4

[tex]x=\frac{1}{4}[/tex]

[tex]\text{Log}_aa=4(\frac{1}{4})[/tex][tex]\text{Log}_aa=1[/tex]

multiply and simplify (5x−4√5)(5x+4√5)

Answers

Answer::

[tex]25x^2-80[/tex]

Explanation:

Given the product:

[tex]\left(5x−4\sqrt{5}\right)\left(5x+4\sqrt{5}\right)[/tex]

First, expand the brackets:

[tex]\begin{gathered} =5x\left(5x+4\sqrt{5}\right)−4\sqrt{5}\left(5x+4\sqrt{5}\right) \\ =(5x)^2+20x\sqrt{5}-20x\sqrt{5}-(4\sqrt{5})^2 \\ =(5x)^2-(4\sqrt{5})^2 \end{gathered}[/tex]

We then simplify:

[tex]\begin{gathered} =5^2x^2-4^2\sqrt{5}^2 \\ =25x^2-16(5) \\ =25x^2-80 \end{gathered}[/tex]

The simplified form of the product is:

[tex]25x^2-80[/tex]

can you help me with the 4th question which is marked b

Answers

we are given the following equation:

[tex]3x+2y=12[/tex]

The slope-intercept form is the following:

[tex]y=mx+b[/tex]

Therefore, we need to solve for "y" in the equation. To do that we will subtract "3x" to both sides:

[tex]\begin{gathered} 3x-3x+2y=12-3x \\ 2y=12-3x \end{gathered}[/tex]

Now we will divide both sides by "2":

[tex]y=\frac{12-3x}{2}[/tex]

Now we will separate the numerator:

[tex]y=\frac{12}{2}-\frac{3}{2}x[/tex]

simplifying:

[tex]y=6-\frac{3}{2}x[/tex]

And thus we get the slope-intercept form.

select the expression that will calculate how many eighths are in 2 bars

Answers

Answer:

Explanations:

Solve the equation for y in terms of x. In other words, algebraicallyrearrange the equation so that the y variable is by itself one side of theequation. Type your answer in the form y = mx + b. If you have a valuethat is not an integer then type it rounded to the nearest hundredth. Donot put spaces between your characters.4x + 2y = 8y = ?

Answers

We can determine an expression of y in terms of x by isolating y on one side of the equation, we can do this by means of some algebraic operations to get:

4x + 2y = 8

1. Subtract 4x from both sides of the equation:

4x - 4x + 2y = 8 - 4x

0 + 2y = 8 - 4x

2y = 8 - 4x

2. Divide both sides by 2

2y/2 = (8 - 4x)/2

y = 4 - 2x

y = -2x + 4

Then, the equation of y in terms of x is y=-2x+4

AC⌢ =84 ∘ , find m∠ADC.

Answers

The measure of minor arc is 84 degree

The expression for the an angle inscribed in a circle, then the measurement of the angle is equal to the half of the measure of its intercepted arc.

[tex]\text{Angle}=\frac{1}{2}m(arc)[/tex]

here we have, arc length = 84 degree

[tex]\begin{gathered} m\angle ADC=\frac{1}{2}(mAC) \\ m\angle ADC=\frac{1}{2}\times84 \\ m\angle ADC=42^o \end{gathered}[/tex]

Angle = 42 degree

Consider the line y=2x/3 - 7 Find the equation of the line that is perpendicular to this line and passes through the point (2, 6)Find the equation of the line that is parallel to this line and passes through the point (2, 6)Equation of perpendicular line: Equation of Parallel line:

Answers

The equation of a line in the slope intercept form is expressed as

y = mx + c

where

m = slope

c = y intercept

The given equation is

y = 2x/3 - 7

By comparing both equations,

m = 2/3

If two lines are perpendicular, it means that the slope of one line is equal to the negative reciprocal of the slope of the other line. This means that the slope of the perpendicular line passing through the point (2, 6) is the negative reciprocal of 2/3. It is - 3/2

Thus, m = - 3/2

We would find the y intercept of the perpendicular line by substituting m = - 3/2, x = 2 and y = 6 into the slope intercept equation. We have

6 = - 3/2 * 2 + c

6 = - 3 + c

c = 6 + 3 = 9

By substituting m = - 3/2 and c = 9 into the slope intercept equation, the equation of the perpendicular line is

y = - 3x/2 + 9

Also,

If two lines are parallel, it means that the slope of one line is equal to the slope of the other line. This means that the slope of the parallel line passing through the point (2, 6) is 2/3

Thus, m = 2/3

We would find the y intercept of the perpendicular line by substituting m = 2/3, x = 2 and y = 6 into the slope intercept equation. We have

6 = 2/3 * 2 + c

6 = 4/3 + c

c = 6 - 4/3 = 14/3

By substituting m = 2/3 and c = 14/3 into the slope intercept equation, the equation of the parallel line is

y = 2x/3 + 14/3

Look for a pattern in the following list. Then use this pattern to predict thenext number. 2, -2, 3, -3, 4, ... *

Answers

Here, we are given the following numbers:

2, -2, 3, -3, 4.........

The pattern here is that a positive integer is followed by its negative value.

We can see that the number after 2 is its negative value -2

The number after 3 is its negative vaule -3

The number after 4 will be its negative which is -4

ANSWER:

-4

Please help me to select the correct image for the representation of the function f(x) = 4 x3x?

Answers

Answer:

Explanation:

Given the below exponential function;

[tex]f(x)=4\cdot3^x[/tex]

To be able to graph the above function, we'll go ahead and choose different values for x and determine the corresponding values of f(x).

When x = 0, we'll have;

[tex]f(0)=4\cdot3^0=4\cdot1=4[/tex]

Looking at all the given four graphs, we can observe that only one of them has a y-interce

The formula for the volume of a rectangular prism is found by multiplying the width, length, and height of the prism. In other words, V = lwh. Solve the formula for the width, w.

Answers

The formula for the volume of a rectangular prism is

[tex]V=l\cdot h\cdot w[/tex]

You need to write the formula for w, note that the width is being miltiplied by "lh"

to cancel this multiplication you have to divide it by "lh" and to keep the equality valid, what is done to one side of the expression must be done to the other, so divide V by "lh" too

[tex]w=\frac{V}{lh}[/tex]

peter is paid k500.00 for the work in 18 hours. how much would he be paid if he had worked six hours

Answers

Given:

500 Kina for 18 hours of work

To determine the amount of payment if he had worked for 6 hours, we use ratio.

So,we let x be the amount of payment for 6 hours of work:

[tex]\begin{gathered} \frac{500\text{ Kina}}{18\text{ hours}}=\frac{x}{6\text{ hours}} \\ \text{Simplify and rearrange} \\ x=\frac{500(6)}{18} \\ \text{Calculate} \\ x=166.67\text{ Kina} \end{gathered}[/tex]

Therefore, he would be paid 166.67 Kina if he had worked for six hours.

The cubic function f(x) = z3 - 6x2 + 11x - 6 has a root at z = 3. a What are the other roots of the function?O r = 3, x = 2O r =-3, x = -2O x = -1, x = -2O x = 1, r = 2

Answers

Given the function f(x) as follows:

[tex]f\mleft(x\mright)=x^3-6x^2+11x-6[/tex]

The function has a root at x = 3

We will use the synthetic division to find the other roots:

We will divide the coefficients by 3

As follows:

So, the given function will be written as follows:

[tex]f(x)=(x-3)(x^2-3x+2)[/tex]

Factor the term of the quadratic function

[tex]f(x)=(x-3)(x-2)(x-1)[/tex]

So, there are three zeros x = 1, 2, 3

So, the answer will be option 4) x = 1, x = 2

32. What is the rate of change of y with the respect to x for 24x - 4y = 50

Answers

The equation for the graph is given as

[tex]24x-4y=50[/tex]

Let us rearrange the equation into its Slope-Intercept form given as

[tex]y=mx+c[/tex]

Where

m = rate of change

c = y-intercept

Therefore, we will have

[tex]-4y=-24x+50[/tex]

Divide all terms by -4 to make y a standalone variable:

[tex]\begin{gathered} \frac{-4y}{-4}=\frac{-24x}{-4}+\frac{50}{(-4)} \\ y=6x-\frac{25}{2} \end{gathered}[/tex]

Comparing with the Slope-Intercept equation, the rate of change is given as 6.

A phone company offers two monthly charge plans. In Plan A, the customer pays a monthly fee of $30 and then an additional 9 cents per minute of use. In PlanB, the customer pays a monthly fee of $33.60 and then an additional 8 cents per minute of use.For what amounts of monthly phone use will Plan A cost less than Plan B?Use m for the number of minutes of phone use, and solve your Inequality for m.

Answers

Let m denote the number of minutes.

Plan A:

The customer pays a monthly fee of $30 and then an additional 9 cents per minute of use.

Mathematically,

[tex]30+0.09m[/tex]

Plan B:

The customer pays a monthly fee of $33.60 and then an additional 8 cents per minute of use.

Mathematically,

[tex]33.60+0.08m[/tex]

For what amounts of monthly phone use will Plan A cost less than Plan B?

[tex]30+0.09m<33.60+0.08m[/tex]

Let us solve the above inequality for m

[tex]\begin{gathered} 30+0.09m<33.60+0.08m \\ 0.09m-0.08m<33.60-30 \\ 0.01m<3.60 \\ m<\frac{3.60}{0.01} \\ m<360 \end{gathered}[/tex]

This means that for less than 360 minutes, plan A will cost less than Plan B.

Please see attachment for question.Fill in the table and then graph the function

Answers

ANSWER

EXPLANATION

First, we have to fill in the table. To do so, we will plug the x-values into the function to find the corresponding value of y,

[tex]\begin{cases}y=-3\cdot3^{-3}=-\frac{3}{3^3}=-\frac{3}{27}=-\frac{1}{9} \\ \\ y=-3\cdot3^{-2}=-\frac{3}{3^2^{}}=-\frac{3}{9}=-\frac{1}{3} \\ \\ y=-3\cdot3^{-1}=-\frac{3}{3^1}=-\frac{3}{3}=-1 \\ \\ y=-3\cdot3^0=-3\cdot1=-3 \\ \\ y=-3\cdot3^1=-3\cdot3=-9 \\ \\ y=-3\cdot3^2=-3\cdot9=-27 \\ \\ y=-3\cdot3^3=-3\cdot27=81\end{cases}[/tex]

So, the table is,

Next, we have to graph all of these points in the coordinate plane. The last one cannot be graphed because y = -81 does not fit in the given coordinate plane. Also, the first two values won't be very accurate because of the scale of the y-axis. The graphed points are,

And finally, to graph the function we join the dots with a line.

Please do this fast and quick I need to sleep

Answers

Answer: [tex]\begin{gathered} Quadratic\text{ equation: }4.9t^2\text{ - 9.8t - 73.5} \\ The\text{ time it will take the cannonball to reach sea level is 5 seconds} \end{gathered}[/tex]

Explanation:

Given:

distance from sea level to top of hill = initial heeight = 73.5 meters

velocity = 9.8 m/s

[tex]\begin{gathered} For\text{ vertical movement:} \\ Final\text{ height = acceleration\lparen t}^2)\text{ + velocity\lparen t\rparen+ initial height} \\ Since\text{ it is reaching sea level, final height = 0} \\ acceleration\text{ = -9.8 m/s}^2 \\ \\ 0\text{ = -}\frac{1}{2}(9.8)t^2\text{ + 9.8t + 73.5m} \end{gathered}[/tex][tex]\begin{gathered} 0\text{ = -4.9t}^2\text{ + 9.8t + 73.5} \\ 4.9t^2\text{ - 9.8t - 73.5 = 0 \lparen quadratic equation\rparen} \\ \\ \text{Using formula method to find the value of t:} \\ t\text{ = }\frac{-b\pm\sqrt[]{b^2-4ac}}{2a} \\ t\text{ = }\frac{-(-9.8)\pm\sqrt{(-9.8)^2-4(4.9)(-73.5)}}{2(4.9)} \\ \text{ t = }\frac{-(-9.8)\pm\sqrt{1536.64}}{9.8} \\ t\text{ = }\frac{9.8\pm39.2}{9.8} \end{gathered}[/tex][tex]\begin{gathered} t\text{ = }\frac{9.8+39.2}{9.8}\text{ ot }\frac{9.8\text{ - 39.2}}{9.8} \\ \\ t\text{ = 5 or -3} \end{gathered}[/tex]

Since we can't have t to be negative, t = 5

The cannonball will reach the sea level at 5 seconds

Which exponential expressions are equivalent to the one below? Check allthat apply.(3.7) 10A. 310 + 710B. (3:7)10O .C. 2110O d. 310.710

Answers

Given the exponential expression:

[tex](3\cdot7)^{10}[/tex]

The equivalent expressions are:

[tex]\begin{gathered} (3\cdot7)^{10}=3^{10}\cdot7^{10} \\ (3\cdot7)^{10}=21^{10} \end{gathered}[/tex]

So, the answer will be options C, D

Ms. Wong wrote a test. Part A had true/false questions, each worth 7 points. Part B had multiple choice questions, each worth 3 points. She made the number of points for Part A equal the number of points for Part B. It was the least number of points for which this was possible.
Answer the following questions.

How many points was each part worth?

How many questions did Part A have?

How many questions did Part B have?

Answers

Considering the least common multiple of 7 and 3, it is found that:

Each part was worth 21 points.Part A had 3 questions.Part B had 7 questions;

How to obtain the measures?

The amount of points of each question in each part are given as follows:

Part A: 7 points.Part B: 3 points.

Both parts have the same number of points, and this amount was the least number of points for which this was possible, hence this amount is the least common multiple of 7 and 3.

Both 3 and 7 are prime numbers, hence the least common multiple of 3 and 7 is given by their multiplication, as follows:

7 x 3 = 21.

Hence each part of the test was worth 21 points.

The number of questions of each part is given by the division of 21 and the worth of each question, hence:

Part A: 21/7 = 3 questions.Part B: 21/3 = 7 questions.

More can be learned about the least common multiple at https://brainly.com/question/10749076

#SPJ1

A- what is R(300) interpret this result B- what is the revenue from the sale of 2,000 hats write in functional notation3 part question

Answers

Okay, here we have this:

Considering the provided information, and the given function we are going to calculate R(300) and then we will interpret the result, so we obtain the following:

[tex]\begin{gathered} R(x)=17x \\ R(300)=17\cdot300 \\ R(300)=5100 \end{gathered}[/tex]

Considering that x corresponds to the number of hats sold, then it means that if 300 hats are sold, the total revenue will be equal to $5100.

I need the steps on how to go about this

Answers

Answer:

Explanation:

Using the slope formula, find the slope of the line through the given points.(-3,-7) and (8,-7)

Answers

the slope of the line is 0

Explanation

The slope of a line is a measure of its steepness of a line , The vertical change between two points is called the rise, and the horizontal change is called the run. The slope equals the rise divided by the run:

[tex]\begin{gathered} slope=\frac{rise}{run}=\frac{change\text{ in y}}{change\text{ in x}}=\frac{y_2-y_1}{x_2-x_1} \\ where \\ P1(x_1,y_1\text{ \rparen and P2\lparen x}_2,y_2)\text{ are 2 points from the line} \end{gathered}[/tex]

so

Step 1

given

[tex]\begin{gathered} P1=(-3,-7) \\ P2=(8,-7) \end{gathered}[/tex]

replace in the formula

[tex]\begin{gathered} slope=\frac{y_{2}-y_{1}}{x_{2}-x_{1}} \\ slope=\frac{-7-(-7)}{8-(-3)}=\frac{-7+7}{11}=\frac{0}{11}=0 \end{gathered}[/tex]

hence, the slope of the line is 0

I hope this helps you

Calculate the quotient below and give your answer in scientific notation.0.000655 x 10-2= ?

Answers

[tex]\frac{0.00065}{5\cdot10^{-2}}[/tex]

Joshua has $1.20 worth of nickels and dimes. He has 6 more nickels than dimes.
Graphically solve a system of equations in order to determine the number of nickels,
x, and the number of dimes, y, that Joshua has.
40
38
36
34
32
30
28
26
24
22
20
18
16
14
12
10
8
0
Click twice to plot each line. Click a line to delete it.
2 4 6 8 10 12 14 16 18 20 22 24 26 28 30 32 34 36 38 40

Answers

Answer:

12 nickels, 6 dimes

Step-by-step explanation:

0.05x + 0.1y = 1.20
x - 6 = y

0.05x + 0.1(x-6) = 1.20
0.05x + 0.1x - 0.6 = 1.20
0.15x = 1.80
x = 12

(12) - 6 = y
y = 6

Now graph y = x - 6 and y = (-1/2)x + 12

If you don't know how to graph the functions, then go to khan academy for help.

I am doing an equation trying to figure out a formula for the volume of a box and I am so lost I will include a picture

Answers

The volume of any rectangular box is expressed as:

[tex]\text{Volume}=\text{length}\times\text{breadth}\times height[/tex]

Now, for the box that will be formed from the figure shown in the question, we will have:

length = 37 - 2x

breadth = 37 - 2x

height = x

Thus, we have that:

[tex]\begin{gathered} \text{Volume}=\text{length}\times\text{breadth}\times height \\ \Rightarrow\text{Volume}=(37-2x)\times(37-2x)\times x \end{gathered}[/tex]

We now simplify the above as:

[tex]\begin{gathered} \text{Volume}=(37-2x)\times(37-2x)\times x \\ \Rightarrow\text{Volume}=(1369-148x+4x^2)\times x \\ \Rightarrow\text{Volume}=1369x-148x^2+4x^3 \\ \Rightarrow\text{ V(x)}=1369x-148x^2+4x^3 \end{gathered}[/tex]

Now that we have obtained the expression for the volume of the box, we now have to find the value of x that maximizes it.

This is done as follows:

Method

- Differentiate the function V(x) with respect to x, and equate to zero as follows:

[tex]\begin{gathered} \Rightarrow V^1\text{(x)}=1369-296x^{}+12x^2 \\ \text{Equating to zero:} \\ 1369-296x^{}+12x^2=0 \\ \text{The roots of the equation are:} \\ \Rightarrow x=6.167\text{ and x = }18.5 \end{gathered}[/tex]

Now we have to find the second derivative of V(x) in order to confirm which value of x makes the function V(x) a maximum

Thus:

[tex]\begin{gathered} \Rightarrow V^{11}\text{(x)}=-296^{}+24x^{} \\ \text{when x = 6.167} \\ \Rightarrow V^{11}\text{(6.167)}=-296^{}+24(6.167)=-296+148.008=-148 \\ \text{when x = }18.5 \\ \Rightarrow V^{11}\text{(18.5)}=-296^{}+24(18.5)=-296+444=148 \end{gathered}[/tex]

Now since the second derivative is a negative number when x = 6.167, we now know for sure that it is that value of x that maximizes the function V(x), and not x = 18.5.

Thus, we can conclude that the value of x that maximizes the volume of the box is:

x = 6.17 inches (to 2 decimal places)

If we had been asked to find the value of x that minimizes the volume, the answer will have been x = 18.5, because this value of x made the second derivative of V(x) positive.

Now, the maximum volume of the box is obtained by simply substituting the value of x that maximizes the function into the original expression for V(x), as follows:

[tex]\begin{gathered} V(x)=1369x-148x^2+4x^3 \\ \text{when x= 6.167} \\ \Rightarrow\text{ V(6.167)}=1369(6.167)-148(6.167)^2+4(6.167)^3 \\ \Rightarrow\text{ V(6.167)}=8442.623-5628.720+938.171 \\ \Rightarrow\text{ V(6.167)}=3752.074in^3 \\ \Rightarrow\text{ V(6.167)}=3752.07in^3\text{ (to 2 decimal places)} \end{gathered}[/tex]

Solve equation 1/4 + 1/7=1/t for t to find the number of days it would take them to paint the house if they worked together. Number 361

Answers

ANSWER:

2.5 days.

STEP-BY-STEP EXPLANATION:

We have the following equation:

[tex]\frac{1}{4}+\frac{1}{7}=\frac{1}{t}[/tex]

We solve for t:

[tex]\begin{gathered} \frac{1\cdot7+4\cdot1}{4\cdot7}=\frac{1}{t} \\ \frac{11}{28}=\frac{1}{t} \\ t=\frac{28}{11}\approx2.5\text{ days} \end{gathered}[/tex]

Therefore, if they work together, they could paint the house in about 2.5 days.

Other Questions
Donovan has 5 times as many as red fish as he does blue fish and half as many gray fish as blue fish. How many total fish (F) does Donovan have?answer optionsF = 6x + 1/2xF = 1/2x + 5 + xF = 6 + 1/2xF = 5x + 1/2x QuesQuestion 3 (1 point)Let sin (47) = 0.7314. Enter an angle (B), in degrees, where cos (B) = 0.7314.Answer:Blank 1:Next PageBack I'll send pic of equation Which of the following is the value of y that solves the system of equations shown below?(1) -5(3) 63x + 4y = 16(2) 8(4) -2x + y =16 Is (5, 8) a solution to this system of equations?X = 52x + y = 18yesnoNeed answer asap a student rolls a marble off of a 1.0 m tall table and it lands 1.5 m away Calculate the initial horizontal velocity. (closest to) Answer A 0.30 m/s B 0.54 m/s C 1.82 m/s D 3.33 m/s If LM = 14 and MN = 11, what is KM?Write your answer as a whole number or as a decimal rounded to the nearest hundredth Write the expression in simplest form.[tex] \sqrt{49 {x}^{5} } [/tex] please help will give you upvotes how do I find the linear change The one-to-one functions g and h are defined as follows.g(x) = 4x - 3h={(-6, 3), (-4, 7), (3, -8), (6, 4)Find the following Paul is driving his car. At the end of two hours he has driven 68 miles, then at the end of 4 hours he has driven 132 miles. What is his average rate of change?_(blank)_miles per hourType your numerical answer below. hat would her speed be at 440 mm ? (in reality, the air resistance will restrict her speed to perhaps 150 km/hkm/h .) express your answer to two significant figures and include the appropriate units. According to one study, an average payout for slots machines is 84 cents on each dollar. What is the percent return on every dollar spent in playing slots?The percent return on every dollakspent in playing slots is %. 6(x-1)=3x+6+3x what is the solution?Many solution No solution or One solution The length of a rectangular rug is 4 less than twice its width. The perimeter of the rug is 34 feet. What is the area? What is the domain of the function shown below? f(x) = log, (x-3) O A. All real numbers greater than 0 O B. All real numbers O c. All real numbers greater than 3 D. All real numbers greater than or equal to -2 Figure DEFG is a parallelogram.If m hey, can someone please help me? i really need it! What was the original cause of united states soldiers killing approximately 300 american indiansmostly unarmed women, children, and older menat wounded knee?.